3
$\begingroup$

I'm working on a number theory problem about deficient numbers and would appreciate some guidance. The exercise is:

Prove that every power of 2 is slightly deficient.

I understand that a number $n$ is called deficient if $\sigma(n) < 2n$, where $\sigma(n)$ is the sum of all divisors of $n$ (including 1 and $n$ itself).

A number is called slightly deficient if $\sigma(n) = 2n - 1$.

For a power of 2, say $n = 2^k$ where $k \geq 1$, the divisors are $1, 2, 2^2, \ldots, 2^k$.

So I need to calculate: $$\sigma(2^k) = 1 + 2 + 2^2 + \cdots + 2^k$$

This is a geometric series, which gives: $$\sigma(2^k) = \frac{2^{k+1} - 1}{2 - 1} = 2^{k+1} - 1$$

For $2^k$ to be slightly deficient, I need to verify that: $$\sigma(2^k) = 2 \cdot 2^k - 1 = 2^{k+1} - 1$$

This seems to match! So my calculation appears to prove the result.

How can I write a proper proof on this statement?

Any feedback on making this proof more rigorous would be appreciated!

$\endgroup$
3
  • 5
    $\begingroup$ You just wrote the proof. $\endgroup$ Commented Oct 5 at 2:38
  • $\begingroup$ This is the proof of powers of $2$ being almost perfect! $\endgroup$ Commented Oct 5 at 3:07
  • $\begingroup$ This problem seems to have a "standard" proof and could be stated as follows: "Prove that $2$ is the only prime $p$ such that every power of $p$ is s.d." or also "What are the primes $p$ such that every power $p^n$ is s.d." $\endgroup$ Commented Oct 5 at 12:56

1 Answer 1

-1
$\begingroup$

By the Fundamental Theorem of Arithmetic, the positive divisors of $2^k$ are exactly

$$ 1,2,2^2, \ldots, 2^k, $$

so $2^i$ for $0 \leq i \leq k$. Therefore $$ \sigma\left(2^k\right)=\sum_{i=0}^k 2^i . $$

Let $S=\sum_{i=0}^k 2^i$. Then $2 S=\sum_{i=1}^{k+1} 2^i=\left(\sum_{i=0}^k 2^i\right)+2^{k+1}-1=S+2^{k+1}-1$. Hence

$$ S=2^{k+1}-1, $$

so

$$ \sigma\left(2^k\right)=2^{k+1}-1=2 \cdot 2^k-1=2 n-1 . $$

Thus $2^k$ is slightly deficient.

$\endgroup$
2
  • 2
    $\begingroup$ How does your proof differ from the one written by the OP? $\endgroup$ Commented Oct 5 at 5:24
  • $\begingroup$ how does the q on makng this rigorous make the answer less helpful? $\endgroup$ Commented Oct 5 at 5:36

You must log in to answer this question.

Start asking to get answers

Find the answer to your question by asking.

Ask question

Explore related questions

See similar questions with these tags.